matheraum.de
Raum für Mathematik
Offene Informations- und Nachhilfegemeinschaft

Für Schüler, Studenten, Lehrer, Mathematik-Interessierte.
Hallo Gast!einloggen | registrieren ]
Startseite · Forum · Wissen · Kurse · Mitglieder · Team · Impressum
Forenbaum
^ Forenbaum
Status Mathe
  Status Schulmathe
    Status Primarstufe
    Status Mathe Klassen 5-7
    Status Mathe Klassen 8-10
    Status Oberstufenmathe
    Status Mathe-Wettbewerbe
    Status Sonstiges
  Status Hochschulmathe
    Status Uni-Analysis
    Status Uni-Lin. Algebra
    Status Algebra+Zahlentheo.
    Status Diskrete Mathematik
    Status Fachdidaktik
    Status Finanz+Versicherung
    Status Logik+Mengenlehre
    Status Numerik
    Status Uni-Stochastik
    Status Topologie+Geometrie
    Status Uni-Sonstiges
  Status Mathe-Vorkurse
    Status Organisatorisches
    Status Schule
    Status Universität
  Status Mathe-Software
    Status Derive
    Status DynaGeo
    Status FunkyPlot
    Status GeoGebra
    Status LaTeX
    Status Maple
    Status MathCad
    Status Mathematica
    Status Matlab
    Status Maxima
    Status MuPad
    Status Taschenrechner

Gezeigt werden alle Foren bis zur Tiefe 2

Navigation
 Startseite...
 Neuerdings beta neu
 Forum...
 vorwissen...
 vorkurse...
 Werkzeuge...
 Nachhilfevermittlung beta...
 Online-Spiele beta
 Suchen
 Verein...
 Impressum
Das Projekt
Server und Internetanbindung werden durch Spenden finanziert.
Organisiert wird das Projekt von unserem Koordinatorenteam.
Hunderte Mitglieder helfen ehrenamtlich in unseren moderierten Foren.
Anbieter der Seite ist der gemeinnützige Verein "Vorhilfe.de e.V.".
Partnerseiten
Dt. Schulen im Ausland: Mathe-Seiten:Weitere Fächer:

Open Source FunktionenplotterFunkyPlot: Kostenloser und quelloffener Funktionenplotter für Linux und andere Betriebssysteme
StartseiteMatheForenIntegrationuneigentl. integral
Foren für weitere Schulfächer findest Du auf www.vorhilfe.de z.B. Geschichte • Erdkunde • Sozialwissenschaften • Politik/Wirtschaft
Forum "Integration" - uneigentl. integral
uneigentl. integral < Integration < Funktionen < eindimensional < reell < Analysis < Hochschule < Mathe < Vorhilfe
Ansicht: [ geschachtelt ] | ^ Forum "Integration"  | ^^ Alle Foren  | ^ Forenbaum  | Materialien

uneigentl. integral: Konvergenz
Status: (Frage) beantwortet Status 
Datum: 12:49 So 18.07.2010
Autor: dr_geissler

Aufgabe
Überprüfen Sie folgende uneigentliche Integrale auf Konvergenz:

[mm] $\integral_{0}^{\infty}{\bruch{sin^2(x)}{x^2}}$ [/mm]

Ich muss doch jetzt die Stammfunktion bilden von

[mm] $\integral_{0}^{R}{\bruch{sin^2(x)}{x^2}}$ [/mm] und dann [mm] &R\to\infty$ [/mm] streben lassen und schauen wie der Grenzwert ist, oder??


Ich hab die Stammfunktion mal versucht herauszubekommen und mein Ergebnis ist:

[mm] $\integral_{0}^{R}{\bruch{sin^2(x)}{x^2}}$=\left[\bruch{sin^2(x)}{x}+c\right]_{0}^{R}$ [/mm]

Sollte das stimmen, kann ich $0$ nicht einsetzen.
Aber für x>0 konvergiert es.

Wie mach ich das jetzt??

        
Bezug
uneigentl. integral: Antwort
Status: (Antwort) fertig Status 
Datum: 12:55 So 18.07.2010
Autor: Teufel

Hi!

Also die Stammfunktion stimmt leider nicht, kannst ja nochmal schauen, ob du den Fehler findest oder gegebenenfalls deinen Rechenweg hier schreiben.
Ansonsten würde ich das so machen:

Das uneigentliche Integral konvergiert genau dann, wenn die dazugehörige Reihe konvergiert, also [mm] \summe_{k=1}^{\infty}\bruch{sin^2(k)}{k^2}. [/mm] Kommst du damit klar?

[anon] Teufel

Bezug
                
Bezug
uneigentl. integral: Frage (beantwortet)
Status: (Frage) beantwortet Status 
Datum: 13:36 So 18.07.2010
Autor: dr_geissler

Ja, dass hilft mir.

Ich versuch es gerade mal. Und zwar mit dem Vergleichskriterium.

[mm] $sin(k)\le [/mm] 1$

[mm] $\bruch{sin(k)}{k}\le\bruch{1}{k}$ [/mm]
[mm] $\left(\bruch{sin(k)}{k}\right)^2\le \left(\bruch{1}{k}\right)^2$ [/mm]
[mm] $\bruch{sin^2(k)}{k^2}\le \bruch{1}{k^2} [/mm]
[mm] \summe_{k=1}^{n}\bruch{sin^2(k)}{k^2} \le \summe_{k=1}^{n}\bruch{1}{k^2}$ [/mm]

geht das so???
oder hab ich da wieder was falsch??

Bezug
                        
Bezug
uneigentl. integral: Antwort
Status: (Antwort) fertig Status 
Datum: 13:50 So 18.07.2010
Autor: Teufel

Genau, so kann man es machen. Nach dem Majorantenkriterium konvergiert dann deine Reihe (absolut) und daher auch das dazugehörige Integral. Zumindest kannst du das machen, wenn ihr den Satz dazu hattet.

[anon] Teufel

Bezug
                                
Bezug
uneigentl. integral: Frage (beantwortet)
Status: (Frage) beantwortet Status 
Datum: 14:07 So 18.07.2010
Autor: dr_geissler

Wir hatten den Satz leider nicht.

Nur etwas mit Potenzreihen und Konvergenzradius.

Ich hab mir aber den Satz und den Beweis rausgeschrieben. Ich beweis das eben in der Klausur und dann kann ich das auch benutzen.

Nur eine Frage dazu. In dem Satz ist von uneigentlichen Integralen
[mm] \integral_{1}^{\infty}{f(x) dx} [/mm] die Rede und nicht von [mm] \integral_{0}^{\infty}{f(x) dx}. [/mm]

Stoß ich da nicht auf ein Problem?

z.B. weil die [mm] \summe_{k=0}^{\infty}\bruch{1}{k^2} [/mm] nicht definiert ist.

Bezug
                                        
Bezug
uneigentl. integral: Antwort
Status: (Antwort) fertig Status 
Datum: 14:15 So 18.07.2010
Autor: M.Rex

Hallo

> Wir hatten den Satz leider nicht.
>  
> Nur etwas mit Potenzreihen und Konvergenzradius.
>  
> Ich hab mir aber den Satz und den Beweis rausgeschrieben.
> Ich beweis das eben in der Klausur und dann kann ich das
> auch benutzen.
>  
> Nur eine Frage dazu. In dem Satz ist von uneigentlichen
> Integralen
> [mm]\integral_{1}^{\infty}{f(x) dx}[/mm] die Rede und nicht von
> [mm]\integral_{0}^{\infty}{f(x) dx}.[/mm]
>  
> Stoß ich da nicht auf ein Problem?

Das ist in der Tat ein Problem. Dann ist es sinnvoll, das Intergral aufzuspalten, wie es schachuzipus in seiner Antwort getan hat..
Der Weg ist definitiv eleganter.

>  
> z.B. weil die [mm]\summe_{k=0}^{\infty}\bruch{1}{k^2}[/mm] nicht
> definiert ist.

Marius


Bezug
                
Bezug
uneigentl. integral: Mitteilung
Status: (Mitteilung) Reaktion unnötig Status 
Datum: 15:00 So 18.07.2010
Autor: Marcel

Hallo Teufel,

> Hi!
>  
> Also die Stammfunktion stimmt leider nicht, kannst ja
> nochmal schauen, ob du den Fehler findest oder
> gegebenenfalls deinen Rechenweg hier schreiben.
>  Ansonsten würde ich das so machen:
>  
> Das uneigentliche Integral konvergiert genau dann, wenn die
> dazugehörige Reihe konvergiert, also
> [mm]\summe_{k=1}^{\infty}\bruch{sin^2(k)}{k^2}.[/mm] Kommst du damit
> klar?

das bedingt aber der Kontrolle, dass $x [mm] \mapsto \sin^2(x)/x^2$ [/mm] auch wirklich monoton fällt (ab einem genügend großen $p [mm] \ge [/mm] 0$).
Vgl. etwa []Wiki, Integralkriterium.

P.S.:
Dass [mm] $\sin^2(x)/x^2 \ge [/mm] 0$ für alle $x [mm] \not= [/mm] 0$ ist klar.

Beste Grüße,
Marcel

Bezug
        
Bezug
uneigentl. integral: Antwort
Status: (Antwort) fertig Status 
Datum: 13:53 So 18.07.2010
Autor: schachuzipus

Hallo dr_geissler,

> Überprüfen Sie folgende uneigentliche Integrale auf
> Konvergenz:
>  
> [mm]\integral_{0}^{\infty}{\bruch{sin^2(x)}{x^2}}[/mm]
>  Ich muss doch jetzt die Stammfunktion bilden von
>  
> [mm]$\integral_{0}^{R}{\bruch{sin^2(x)}{x^2}}$[/mm] und dann
> [mm]&R\to\infty$[/mm] streben lassen und schauen wie der Grenzwert
> ist, oder??
>  
>
> Ich hab die Stammfunktion mal versucht herauszubekommen und
> mein Ergebnis ist:
>  
> [mm]$\integral_{0}^{R}{\bruch{sin^2(x)}{x^2}}$=\left[\bruch{sin^2(x)}{x}+c\right]_{0}^{R}$[/mm]
>  
> Sollte das stimmen, kann ich [mm]0[/mm] nicht einsetzen.
>  Aber für x>0 konvergiert es.

Ich finde es eleganter, das Integral aufzuspalten:

[mm] $\int\limits_{0}^{\infty}{\frac{\sin^2(x)}{x^2} \ dx}=\int\limits_{0}^{1}{\frac{\sin^2(x)}{x^2} \ dx}+\int\limits_{1}^{\infty}{\frac{\sin^2(x)}{x^2} \ dx}$ [/mm]

Nun zeige, dass beide Teilintegrale endlich sind:

Für das hintere verwende: [mm] $\sin^2(x)\le [/mm] 1$ und du hast mit [mm] $\int\limits_{1}^{\infty}{\frac{1}{x^2} \ dx}$ [/mm] eine konv, Majorante.

Für das erste beachte, dass sich [mm] $\frac{\sin(x)}{x}$ [/mm] in 0 stetig ergänzen lässt. Wieso und wie?

Also auch [mm] $\\\left(\frac{\sin(x)}{x}\right)^2$ [/mm]

Damit hast du beim ersten Integral einen stetigen Integranden auf dem Kompaktum [0,1], also etwas Beschränktes, also auch ein endliches Integral ...

Fülle mal die paar Lücken ...

>  
> Wie mach ich das jetzt??


Gruß

schachuzipus

Bezug
                
Bezug
uneigentl. integral: Frage (beantwortet)
Status: (Frage) beantwortet Status 
Datum: 14:28 So 18.07.2010
Autor: dr_geissler


> Hallo dr_geissler,
>  
> > Überprüfen Sie folgende uneigentliche Integrale auf
> > Konvergenz:
>  >  
> > [mm]\integral_{0}^{\infty}{\bruch{sin^2(x)}{x^2}}[/mm]
>  >  Ich muss doch jetzt die Stammfunktion bilden von
>  >  
> > [mm]$\integral_{0}^{R}{\bruch{sin^2(x)}{x^2}}$[/mm] und dann
> > [mm]&R\to\infty$[/mm] streben lassen und schauen wie der Grenzwert
> > ist, oder??
>  >  
> >
> > Ich hab die Stammfunktion mal versucht herauszubekommen und
> > mein Ergebnis ist:
>  >  
> >
> [mm]$\integral_{0}^{R}{\bruch{sin^2(x)}{x^2}}$=\left[\bruch{sin^2(x)}{x}+c\right]_{0}^{R}$[/mm]
>  >  
> > Sollte das stimmen, kann ich [mm]0[/mm] nicht einsetzen.
>  >  Aber für x>0 konvergiert es.
>  
> Ich finde es eleganter, das Integral aufzuspalten:
>  
> [mm]\int\limits_{0}^{\infty}{\frac{\sin^2(x)}{x^2} \ dx}=\int\limits_{0}^{1}{\frac{\sin^2(x)}{x^2} \ dx}+\int\limits_{1}^{\infty}{\frac{\sin^2(x)}{x^2} \ dx}[/mm]
>  
> Nun zeige, dass beide Teilintegrale endlich sind:
>  
> Für das hintere verwende: [mm]\sin^2(x)\le 1[/mm] und du hast mit
> [mm]\int\limits_{1}^{\infty}{\frac{1}{x^2} \ dx}[/mm] eine konv,
> Majorante.
>  
> Für das erste beachte, dass sich [mm]\frac{\sin(x)}{x}[/mm] in 0
> stetig ergänzen lässt. Wieso und wie?

Ganz genau weiß ich das nicht.
Ich denke mal, weil [mm] \limes_{n\rightarrow\0}\bruch{sin(x)}{x}=1 [/mm] ist.

und [mm] \limes_{n\rightarrow\0}\bruch{sin^2(x)}{x^2}=1 [/mm]

Ich denke mal, ohne das geprüft zu haben, dass [mm] \limes_{n\rightarrow\0}\bruch{sin^k(x)}{x^k}=1. [/mm]

Denke mal, ich bekomme Probleme wenn [mm] \bruch{sin^l(x)}{x^k} [/mm] mit [mm] $l\not= [/mm] k$.


Mach ich mich soweit gut?


>  
> Also auch [mm]\\\left(\frac{\sin(x)}{x}\right)^2[/mm]
>  
> Damit hast du beim ersten Integral einen stetigen
> Integranden auf dem Kompaktum [0,1], also etwas
> Beschränktes, also auch ein endliches Integral ...
>  
> Fülle mal die paar Lücken ...
>  
> >  

> > Wie mach ich das jetzt??
>
>
> Gruß
>  
> schachuzipus


Bezug
                        
Bezug
uneigentl. integral: Antwort
Status: (Antwort) fertig Status 
Datum: 14:57 So 18.07.2010
Autor: Marcel

Hallo,

> > Hallo dr_geissler,
>  >  
> > > Überprüfen Sie folgende uneigentliche Integrale auf
> > > Konvergenz:
>  >  >  
> > > [mm]\integral_{0}^{\infty}{\bruch{sin^2(x)}{x^2}}[/mm]
>  >  >  Ich muss doch jetzt die Stammfunktion bilden von
>  >  >  
> > > [mm]\integral_{0}^{R}{\bruch{sin^2(x)}{x^2}}[/mm] und dann
> > > [mm]&R\to\infty$[/mm] streben lassen und schauen wie der Grenzwert
> > > ist, oder??
>  >  >  
> > >
> > > Ich hab die Stammfunktion mal versucht herauszubekommen und
> > > mein Ergebnis ist:
>  >  >  
> > >
> >
> [mm]\integral_{0}^{R}{\bruch{sin^2(x)}{x^2}}=\left[\bruch{sin^2(x)}{x}+c\right]_{0}^{R}[/mm]
>  >  >  
> > > Sollte das stimmen, kann ich [mm]0[/mm] nicht einsetzen.
>  >  >  Aber für x>0 konvergiert es.
>  >  
> > Ich finde es eleganter, das Integral aufzuspalten:
>  >  
> > [mm]\int\limits_{0}^{\infty}{\frac{\sin^2(x)}{x^2} \ dx}=\int\limits_{0}^{1}{\frac{\sin^2(x)}{x^2} \ dx}+\int\limits_{1}^{\infty}{\frac{\sin^2(x)}{x^2} \ dx}[/mm]

>

> >  

> > Nun zeige, dass beide Teilintegrale endlich sind:
>  >  
> > Für das hintere verwende: [mm]\sin^2(x)\le 1[/mm] und du hast mit
> > [mm]\int\limits_{1}^{\infty}{\frac{1}{x^2} \ dx}[/mm] eine konv,
> > Majorante.
>  >  
> > Für das erste beachte, dass sich [mm]\frac{\sin(x)}{x}[/mm] in 0
> > stetig ergänzen lässt. Wieso und wie?
>  
> Ganz genau weiß ich das nicht.
>  Ich denke mal, weil
> [mm]\limes_{\red{n\rightarrow\0}}\bruch{sin(x)}{x}=1[/mm] ist.

ja! (Sofern du [mm] $\red{n \rightarrow 0}$ [/mm] bzw. [mm] $\red{n\rightarrow\0}$ [/mm] (auch im Folgenden) durch [mm] $\blue{x \to 0}$ [/mm] ersetzt.)
  

> und [mm]\limes_{n\rightarrow\0}\bruch{sin^2(x)}{x^2}=1[/mm]

Auch das ist korrekt.
  

> Ich denke mal, ohne das geprüft zu haben, dass
> [mm]\limes_{n\rightarrow\0}\bruch{sin^k(x)}{x^k}=1.[/mm]

In der Tat, s.u..
  

> Denke mal, ich bekomme Probleme wenn [mm]\bruch{sin^l(x)}{x^k}[/mm]
> mit [mm]l\not= k[/mm].

Dann kann man ja, wenn z.B. $l < [mm] k\,$ [/mm] ist, schreiben:
[mm] $$\bruch{\sin^l(x)}{x^k}=\frac{1}{x^{k-l}}*\frac{\sin^{l}(x)}{x^l}$$ [/mm]
Das macht in der Tat Probleme (jedenfalls bei $x [mm] \to [/mm] 0$). Aber wenn $l > [mm] k\,$ [/mm] ist, wird das auch sehr einfach (überlege es mal analog).

> Mach ich mich soweit gut?

[ok]
Bis auf den kleinen Fakt: Du sollst nicht raten!
  

Also hier die Überlegungen (weil du es oben angesprochen hast, machen wir es für jedes $k [mm] \in \IN_0$): [/mm]
Ist $k [mm] \in \IN_0\,,$ [/mm] so gilt
[mm] $$\lim_{x \to 0}\frac{\sin^k(x)}{x^k}=\lim_{x \to 0}\left(\frac{\sin(x)}{x}\right)^k\blue{=}\left(\lim_{x \to 0}\frac{\sin(x)}{x}\right)^k=1^k=1\,.$$ [/mm]

Du brauchst dieses Ergebnis aber nur für [mm] $k=2\,.$ [/mm]

Begründe dabei noch das [mm] $\blue{=}\,,$ [/mm] auch, indem Du z.B. mit de l'Hospital
[mm] $$\lim_{x \to 0}\frac{\sin(x)}{x}=1$$ [/mm]
nachweist (oder mit der Potenzreihenentwicklung von [mm] $\sin$ [/mm] um [mm] $x_0=0$), [/mm] und beachte, dass Du bei Dir eigentlich
[mm] $$\lim_{x \to 0^+}=\lim_{\substack{x \to 0\\x > 0}}$$ [/mm]
brauchst. Aber wenn der Grenzwert von [mm] $f\,$ [/mm] an der Stelle [mm] $a\,,$ [/mm] also [mm] $\lim_{x \to a}f(x)$ [/mm] existiert, dann auch der rechtsseitige GW von [mm] $f\,$ [/mm] an der Stelle [mm] $a\,,$ [/mm] also [mm] $\lim_{x \to a^+}f(x)\,,$ [/mm] und beide sind dann gleich.

Beste Grüße,
Marcel

Bezug
                                
Bezug
uneigentl. integral: Frage (beantwortet)
Status: (Frage) beantwortet Status 
Datum: 17:39 So 18.07.2010
Autor: dr_geissler

Wie schreib ich das jetzt?

$ [mm] \int\limits_{0}^{\infty}{\frac{\sin^2(x)}{x^2} \ dx}=\int\limits_{0}^{1}{\frac{\sin^2(x)}{x^2} \ dx}+\int\limits_{1}^{\infty}{\frac{\sin^2(x)}{x^2} \ dx} [/mm] $

da [mm] \summe_{i=1}^{\infty}\frac{\sin^2(x)}{x^2} [/mm] konvergiert, konvergiert auch [mm] \int\limits_{1}^{\infty}{\frac{\sin^2(x)}{x^2} \ dx} [/mm]

da [mm] \limes_{x\rightarrow 0}\frac{\sin^2(x)}{x^2}=1 [/mm] folgt [mm] \int\limits_{0}^{1}{\frac{\sin^2(x)}{x^2} \ dx} [/mm] konvergiert.

Daraus folgt [mm] \int\limits_{0}^{\infty}{\frac{\sin^2(x)}{x^2} \ dx} [/mm] konvergiert.


Kann man das in der Klausur so schreiben??

Bezug
                                        
Bezug
uneigentl. integral: Antwort
Status: (Antwort) fertig Status 
Datum: 17:49 So 18.07.2010
Autor: schachuzipus

Hallo nochmal,

> Wie schreib ich das jetzt?
>  
> [mm]\int\limits_{0}^{\infty}{\frac{\sin^2(x)}{x^2} \ dx}=\int\limits_{0}^{1}{\frac{\sin^2(x)}{x^2} \ dx}+\int\limits_{1}^{\infty}{\frac{\sin^2(x)}{x^2} \ dx}[/mm] [ok]

Ja, siehe oben!

>  
> da [mm]\summe_{i=1}^{\infty}\frac{\sin^2(x)}{x^2}[/mm] konvergiert,
> konvergiert auch
> [mm]\int\limits_{1}^{\infty}{\frac{\sin^2(x)}{x^2} \ dx}[/mm]

Begründung?

Ich hatte doch geschrieben, wie es ganz einfach abzuschätzen ist.

HAst du meine Antwort nicht gelesen?

Dann ist eh alles weitere sinnlos ...

Echt, Mensch, Mensch.

Nochmal:

Es ist [mm] $\sin^2(x)\le [/mm] 1$

Also [mm] $\int\limits_{1}^{\infty}{\frac{\sin^2(x)}{x^2} \ dx} [/mm] \ [mm] \le [/mm] \ [mm] \int\limits_{1}^{\infty}{\frac{1}{x^2} \ dx}=\ldots$ [/mm]

Ausrechnen und zeigen, dass das einen endlichen Wert hat. Welchen?

Mit der obigen Abschätzung ist dann [mm] $\int\limits_{1}^{\infty}{\frac{\sin^2(x)}{x^2} \ dx}\le [/mm] \ [mm] \text{diesem Wert}$, [/mm] also endlich.

Für das Integral von 0 bis 1 ist ja nur 0 ein kritischer Punkt. Aber der Integrand [mm] $f(x)=\frac{\sin^2(x)}{x^2}$ [/mm] lässt sich in 0 stetig ergänzen durch $f(0):=1$

Damit ist $f$ auf dem gesamten geschlossen Intervall $[0,1]$ stetig, also beschränkt durch [mm] $\max\limits_{x\in[0,1]}\{|f(x)|\}$ [/mm]

Damit ist das Integral über das Intervall $[0,1]$ ebenfalls beschränkt.

Warum? ...

>  
> da [mm]\limes_{x\rightarrow 0}\frac{\sin^2(x)}{x^2}=1[/mm] folgt
> [mm]\int\limits_{0}^{1}{\frac{\sin^2(x)}{x^2} \ dx}[/mm]
> konvergiert.
>  
> Daraus folgt [mm]\int\limits_{0}^{\infty}{\frac{\sin^2(x)}{x^2} \ dx}[/mm]
> konvergiert.
>  
>
> Kann man das in der Klausur so schreiben??


So sicher nicht ...

Gruß

schachuzipus

Bezug
                                                
Bezug
uneigentl. integral: Frage (beantwortet)
Status: (Frage) beantwortet Status 
Datum: 18:32 So 18.07.2010
Autor: dr_geissler


> Hallo nochmal,
>  
> > Wie schreib ich das jetzt?
>  >  
> > [mm]\int\limits_{0}^{\infty}{\frac{\sin^2(x)}{x^2} \ dx}=\int\limits_{0}^{1}{\frac{\sin^2(x)}{x^2} \ dx}+\int\limits_{1}^{\infty}{\frac{\sin^2(x)}{x^2} \ dx}[/mm]
> [ok]
>  
> Ja, siehe oben!
>  
> >  

> > da [mm]\summe_{i=1}^{\infty}\frac{\sin^2(x)}{x^2}[/mm] konvergiert,
> > konvergiert auch
> > [mm]\int\limits_{1}^{\infty}{\frac{\sin^2(x)}{x^2} \ dx}[/mm]
>  
> Begründung?

Die hatte ich doch schon in einer anderen Antwort stehen, wollte es ja nur kur machen. Aber nochmal:

Begründung:
$ [mm] sin(k)\le [/mm] 1 $

$ [mm] \bruch{sin(k)}{k}\le\bruch{1}{k} [/mm] $

$ [mm] \left(\bruch{sin(k)}{k}\right)^2\le \left(\bruch{1}{k}\right)^2 [/mm] $

$ [mm] $\bruch{sin^2(k)}{k^2}\le \bruch{1}{k^2} [/mm] $

$ [mm] \summe_{k=1}^{n}\bruch{sin^2(k)}{k^2} \le \summe_{k=1}^{n}\bruch{1}{k^2}$ [/mm] $

>  
> Ich hatte doch geschrieben, wie es ganz einfach
> abzuschätzen ist.
>  
> HAst du meine Antwort nicht gelesen?
>  
> Dann ist eh alles weitere sinnlos ...
>  
> Echt, Mensch, Mensch.
>  
> Nochmal:
>  
> Es ist [mm]\sin^2(x)\le 1[/mm]
>  
> Also [mm]\int\limits_{1}^{\infty}{\frac{\sin^2(x)}{x^2} \ dx} \ \le \ \int\limits_{1}^{\infty}{\frac{1}{x^2} \ dx}=\ldots[/mm]
>  
> Ausrechnen und zeigen, dass das einen endlichen Wert hat.
> Welchen?

<2 (ich glaube [mm] \bruch{\pi^2}{6}) [/mm]

Aber ist das wichtig??
Der Satz heißt doch ...Integral... kob´nvergiert genau dann wenn die dazugehörige Reihe konvergiert.



>  
> Mit der obigen Abschätzung ist dann
> [mm]\int\limits_{1}^{\infty}{\frac{\sin^2(x)}{x^2} \ dx}\le \ \text{diesem Wert}[/mm],
> also endlich.
>  
> Für das Integral von 0 bis 1 ist ja nur 0 ein kritischer
> Punkt. Aber der Integrand [mm]f(x)=\frac{\sin^2(x)}{x^2}[/mm] lässt
> sich in 0 stetig ergänzen durch [mm]f(0):=1[/mm]
>  
> Damit ist [mm]f[/mm] auf dem gesamten geschlossen Intervall [mm][0,1][/mm]
> stetig, also beschränkt durch
> [mm]\max\limits_{x\in[0,1]}\{|f(x)|\}[/mm]
>  
> Damit ist das Integral über das Intervall [mm][0,1][/mm] ebenfalls
> beschränkt.
>  
> Warum? ...

Weil das Integral über einem geschlossen Intevall ein supremum und ein infimum hat. bzw. die Riemannsche Obersumme und Untersumme.

Oder?

>  
> >  

> > da [mm]\limes_{x\rightarrow 0}\frac{\sin^2(x)}{x^2}=1[/mm] folgt
> > [mm]\int\limits_{0}^{1}{\frac{\sin^2(x)}{x^2} \ dx}[/mm]
> > konvergiert.
>  >  
> > Daraus folgt [mm]\int\limits_{0}^{\infty}{\frac{\sin^2(x)}{x^2} \ dx}[/mm]
> > konvergiert.
>  >  
> >
> > Kann man das in der Klausur so schreiben??
>
>
> So sicher nicht ...
>  
> Gruß
>  
> schachuzipus


Jetzt besser??


Bezug
                                                        
Bezug
uneigentl. integral: Antwort
Status: (Antwort) fertig Status 
Datum: 11:29 Mo 19.07.2010
Autor: Marcel

Hallo,

> > Hallo nochmal,
>  >  
> > > Wie schreib ich das jetzt?
>  >  >  
> > > [mm]\int\limits_{0}^{\infty}{\frac{\sin^2(x)}{x^2} \ dx}=\int\limits_{0}^{1}{\frac{\sin^2(x)}{x^2} \ dx}+\int\limits_{1}^{\infty}{\frac{\sin^2(x)}{x^2} \ dx}[/mm]
> > [ok]
>  >  
> > Ja, siehe oben!
>  >  
> > >  

> > > da [mm]\summe_{i=1}^{\infty}\frac{\sin^2(x)}{x^2}[/mm] konvergiert,
> > > konvergiert auch
> > > [mm]\int\limits_{1}^{\infty}{\frac{\sin^2(x)}{x^2} \ dx}[/mm]
>  >

>  
> > Begründung?
>  
> Die hatte ich doch schon in einer anderen Antwort stehen,
> wollte es ja nur kur machen. Aber nochmal:
>  
> Begründung:
>  [mm]sin(k)\le 1[/mm]
>  
> [mm]\bruch{sin(k)}{k}\le\bruch{1}{k}[/mm]
>  
> [mm]\left(\bruch{sin(k)}{k}\right)^2\le \left(\bruch{1}{k}\right)^2[/mm]
>  
> $ [mm]$\bruch{sin^2(k)}{k^2}\le \bruch{1}{k^2}[/mm] $
>  
> $ [mm]\summe_{k=1}^{n}\bruch{sin^2(k)}{k^2} \le \summe_{k=1}^{n}\bruch{1}{k^2}$[/mm]
> $
>  
> >  

> > Ich hatte doch geschrieben, wie es ganz einfach
> > abzuschätzen ist.
>  >  
> > HAst du meine Antwort nicht gelesen?
>  >  
> > Dann ist eh alles weitere sinnlos ...
>  >  
> > Echt, Mensch, Mensch.
>  >  
> > Nochmal:
>  >  
> > Es ist [mm]\sin^2(x)\le 1[/mm]
>  >  
> > Also [mm]\int\limits_{1}^{\infty}{\frac{\sin^2(x)}{x^2} \ dx} \ \le \ \int\limits_{1}^{\infty}{\frac{1}{x^2} \ dx}=\ldots[/mm]
>  
> >  

> > Ausrechnen und zeigen, dass das einen endlichen Wert hat.
> > Welchen?
>  
> <2 (ich glaube [mm]\bruch{\pi^2}{6})[/mm]Eingabefehler: "\left" und "\right" müssen immer paarweise auftreten, es wurde aber ein Teil ohne Entsprechung gefunden (siehe rote Markierung)



wie kommst Du darauf? Es ist doch mit dem HDI sehr sehr einfach:
$$\int_1^\infty 1/x^2\;dx=\lim_{a \to \infty} (\left.\;-1/x\;\right|_{x=1}^{x=a})=\lim_{a \to \infty}-1/a-(-1/1)=1\,.$$
  

> Aber ist das wichtig??
> Der Satz heißt doch ...Integral... kob´nvergiert genau
> dann wenn die dazugehörige Reihe konvergiert.


Ja, aber dann hättest Du auch gleich die ganze Aufgabe mit diesem Satz - den Du auch nochmal in dieser Mitteilung hier verlinkt findest (das sogenannte Integralkriterium) - lösen können. Schachuzipus wollte ja gerade, dass Du "nur" mit Integralen arbeitest. Und da benutzt er ein Analogon, was Du von Reihen kennst: Er sucht eine "Majorante", für die das Integral existiert; sozusagen eine "bzgl. des Integrals konvergente Majorante".  
(Das ganze kann man analog zu []dieser Aussage betrachten.)

>
>
> >  

> > Mit der obigen Abschätzung ist dann
> > [mm]\int\limits_{1}^{\infty}{\frac{\sin^2(x)}{x^2} \ dx}\le \ \text{diesem Wert}[/mm],
> > also endlich.
>  >  
> > Für das Integral von 0 bis 1 ist ja nur 0 ein kritischer
> > Punkt. Aber der Integrand [mm]f(x)=\frac{\sin^2(x)}{x^2}[/mm] lässt
> > sich in 0 stetig ergänzen durch [mm]f(0):=1[/mm]
>  >  
> > Damit ist [mm]f[/mm] auf dem gesamten geschlossen Intervall [mm][0,1][/mm]
> > stetig, also beschränkt durch
> > [mm]\max\limits_{x\in[0,1]}\{|f(x)|\}[/mm]
>  >  
> > Damit ist das Integral über das Intervall [mm][0,1][/mm] ebenfalls
> > beschränkt.
>  >  
> > Warum? ...
>  
> Weil das Integral über einem geschlossen Intevall ein
> supremum und ein infimum hat. bzw. die Riemannsche
> Obersumme und Untersumme.
>
> Oder?
>  
> >  

> > >  

> > > da [mm]\limes_{x\rightarrow 0}\frac{\sin^2(x)}{x^2}=1[/mm] folgt
> > > [mm]\int\limits_{0}^{1}{\frac{\sin^2(x)}{x^2} \ dx}[/mm]
> > > konvergiert.
>  >  >  
> > > Daraus folgt [mm]\int\limits_{0}^{\infty}{\frac{\sin^2(x)}{x^2} \ dx}[/mm]
> > > konvergiert.
>  >  >  
> > >
> > > Kann man das in der Klausur so schreiben??
> >
> >
> > So sicher nicht ...
>  >  
> > Gruß
>  >  
> > schachuzipus
>
>
> Jetzt besser??

Also nochmal das ganze, wie Schachuzipus es meinte:
Es gilt
[mm] $$\blue{(\*)}\;\;\int\limits_{0}^{\infty}{\frac{\sin^2(x)}{x^2} \ dx}=\int\limits_{0}^{1}{\frac{\sin^2(x)}{x^2} \ dx}+\int\limits_{1}^{\infty}{\frac{\sin^2(x)}{x^2} \ dx}\,.$$ [/mm]

Das linke Integral aus [mm] $\blue{(\*)}$ [/mm] existiert jedenfalls dann, wenn die beiden rechts existieren.
Sei [mm] $f(x):=\frac{\sin^2(x)}{x^2}$ [/mm] ($x > 0$) und Korrektur meines [mm] Verschreibers$f(0):=\red{1}\,.$ [/mm] Weil [mm] $f\,$ [/mm] stetig ist, ist [mm] $f_{|[0,1]}\,$ [/mm] als stetige Funktion auf einer kompakten Menge beschränkt, d.h. es gilt (wegen $f [mm] \ge [/mm] 0$) $0 [mm] \le f_{|[0,1]} \le [/mm] M$ mit einer Konstanten $M > [mm] 0\,.$ [/mm]
Somit
[mm] $$\int\limits_{0}^{1}{\frac{\sin^2(x)}{x^2} \ dx} \le \int_0^1 Mdx=M\,,$$ [/mm]
also existiert das erste Integral in [mm] $\blue{(\*)}$ [/mm] rechterhand.

Wegen [mm] $\int\limits_{1}^{\infty}{\frac{\sin^2(x)}{x^2} \ dx} \le \int_1^\infty \frac{1}{x^2}dx=1$ [/mm] (s.o.) folgt auch die Existenz von [mm] $\int\limits_{1}^{\infty}{\frac{\sin^2(x)}{x^2} \ dx}\,,$ [/mm]
d.h. das zweite Integral rechterhand aus [mm] $\blue{(\*)}$ [/mm] existiert, und damit existiert das Integral linkerhand von [mm] $\blue{(\*)}\,.$ [/mm]

Zudem können wir mit den obigen Überlegungen abschätzen:
[mm] $$\int\limits_{0}^{\infty}{\frac{\sin^2(x)}{x^2} \ dx} \le M+1\,,$$ [/mm]
was natürlich auch nochmal insbesondere die Existenz unseres Integrals zeigt.

Beste Grüße,
Marcel

Bezug
                                                                
Bezug
uneigentl. integral: Mitteilung
Status: (Mitteilung) Reaktion unnötig Status 
Datum: 11:34 Mo 19.07.2010
Autor: fred97


> Hallo,
>  
> > > Hallo nochmal,
>  >  >  
> > > > Wie schreib ich das jetzt?
>  >  >  >  
> > > > [mm]\int\limits_{0}^{\infty}{\frac{\sin^2(x)}{x^2} \ dx}=\int\limits_{0}^{1}{\frac{\sin^2(x)}{x^2} \ dx}+\int\limits_{1}^{\infty}{\frac{\sin^2(x)}{x^2} \ dx}[/mm]
> > > [ok]
>  >  >  
> > > Ja, siehe oben!
>  >  >  
> > > >  

> > > > da [mm]\summe_{i=1}^{\infty}\frac{\sin^2(x)}{x^2}[/mm] konvergiert,
> > > > konvergiert auch
> > > > [mm]\int\limits_{1}^{\infty}{\frac{\sin^2(x)}{x^2} \ dx}[/mm]
>  
> >  >

> >  

> > > Begründung?
>  >  
> > Die hatte ich doch schon in einer anderen Antwort stehen,
> > wollte es ja nur kur machen. Aber nochmal:
>  >  
> > Begründung:
>  >  [mm]sin(k)\le 1[/mm]
>  >  
> > [mm]\bruch{sin(k)}{k}\le\bruch{1}{k}[/mm]
>  >  
> > [mm]\left(\bruch{sin(k)}{k}\right)^2\le \left(\bruch{1}{k}\right)^2[/mm]
>  
> >  

> > $ [mm]$\bruch{sin^2(k)}{k^2}\le \bruch{1}{k^2}[/mm] $
>  >  
> > $ [mm]\summe_{k=1}^{n}\bruch{sin^2(k)}{k^2} \le \summe_{k=1}^{n}\bruch{1}{k^2}$[/mm]
> > $
>  >  
> > >  

> > > Ich hatte doch geschrieben, wie es ganz einfach
> > > abzuschätzen ist.
>  >  >  
> > > HAst du meine Antwort nicht gelesen?
>  >  >  
> > > Dann ist eh alles weitere sinnlos ...
>  >  >  
> > > Echt, Mensch, Mensch.
>  >  >  
> > > Nochmal:
>  >  >  
> > > Es ist [mm]\sin^2(x)\le 1[/mm]
>  >  >  
> > > Also [mm]\int\limits_{1}^{\infty}{\frac{\sin^2(x)}{x^2} \ dx} \ \le \ \int\limits_{1}^{\infty}{\frac{1}{x^2} \ dx}=\ldots[/mm]
>  
> >  

> > >  

> > > Ausrechnen und zeigen, dass das einen endlichen Wert hat.
> > > Welchen?
>  >  
> > <2 (ich glaube [mm]\bruch{\pi^2}{6})[/mm]
>  
> wie kommst Du darauf? Es ist doch mit dem HDI sehr sehr
> einfach:
>  [mm]\int_1^\infty 1/x^2\;dx=\lim_{a \to \infty} (\left.\;-1/x\;\right|_{x=1}^{x=a})=\lim_{a \to \infty}-1/a-(-1/1)=1\,.[/mm]
>  
>  
> > Aber ist das wichtig??
> > Der Satz heißt doch ...Integral... kob´nvergiert genau
> > dann wenn die dazugehörige Reihe konvergiert.
>  
>
> Ja, aber dann hättest Du auch gleich die ganze Aufgabe mit
> diesem Satz - den Du auch nochmal
> in dieser Mitteilung hier
> verlinkt findest (das sogenannte Integralkriterium) -
> lösen können. Schachuzipus wollte ja gerade, dass Du
> "nur" mit Integralen arbeitest. Und da benutzt er ein
> Analogon, was Du von Reihen kennst: Er sucht eine
> "Majorante", für die das Integral existiert; sozusagen
> eine "bzgl. des Integrals konvergente Majorante".  
> (Das ganze kann man analog zu
> []dieser Aussage
> betrachten.)
>  >

> >
> > >  

> > > Mit der obigen Abschätzung ist dann
> > > [mm]\int\limits_{1}^{\infty}{\frac{\sin^2(x)}{x^2} \ dx}\le \ \text{diesem Wert}[/mm],
> > > also endlich.
>  >  >  
> > > Für das Integral von 0 bis 1 ist ja nur 0 ein kritischer
> > > Punkt. Aber der Integrand [mm]f(x)=\frac{\sin^2(x)}{x^2}[/mm] lässt
> > > sich in 0 stetig ergänzen durch [mm]f(0):=1[/mm]
>  >  >  
> > > Damit ist [mm]f[/mm] auf dem gesamten geschlossen Intervall [mm][0,1][/mm]
> > > stetig, also beschränkt durch
> > > [mm]\max\limits_{x\in[0,1]}\{|f(x)|\}[/mm]
>  >  >  
> > > Damit ist das Integral über das Intervall [mm][0,1][/mm] ebenfalls
> > > beschränkt.
>  >  >  
> > > Warum? ...
>  >  
> > Weil das Integral über einem geschlossen Intevall ein
> > supremum und ein infimum hat. bzw. die Riemannsche
> > Obersumme und Untersumme.
> >
> > Oder?
>  >  
> > >  

> > > >  

> > > > da [mm]\limes_{x\rightarrow 0}\frac{\sin^2(x)}{x^2}=1[/mm] folgt
> > > > [mm]\int\limits_{0}^{1}{\frac{\sin^2(x)}{x^2} \ dx}[/mm]
> > > > konvergiert.
>  >  >  >  
> > > > Daraus folgt [mm]\int\limits_{0}^{\infty}{\frac{\sin^2(x)}{x^2} \ dx}[/mm]
> > > > konvergiert.
>  >  >  >  
> > > >
> > > > Kann man das in der Klausur so schreiben??
> > >
> > >
> > > So sicher nicht ...
>  >  >  
> > > Gruß
>  >  >  
> > > schachuzipus
> >
> >
> > Jetzt besser??
>  
> Also nochmal das ganze, wie Schachuzipus es meinte:
>  Es gilt
>  
> [mm]\blue{(\*)}\;\;\int\limits_{0}^{\infty}{\frac{\sin^2(x)}{x^2} \ dx}=\int\limits_{0}^{1}{\frac{\sin^2(x)}{x^2} \ dx}+\int\limits_{1}^{\infty}{\frac{\sin^2(x)}{x^2} \ dx}\,.[/mm]
>  
> Das linke Integral aus [mm]\blue{(\*)}[/mm] existiert jedenfalls
> dann, wenn die beiden rechts existieren.
>  Sei [mm]f(x):=\frac{\sin^2(x)}{x^2}[/mm] ([mm]x > 0[/mm]) und [mm]f(0):=0\,.[/mm]




Hallo Marcel,

damit f stetig ist, sollt man setzen:  [mm]f(0):=1\,.[/mm]

Gruß FRED


> Weil [mm]f\,[/mm] stetig ist, ist [mm]f_{|[0,1]}\,[/mm] als stetige Funktion
> auf einer kompakten Menge beschränkt, d.h. es gilt (wegen
> [mm]f \ge 0[/mm]) [mm]0 \le f_{|[0,1]} \le M[/mm] mit einer Konstanten [mm]M > 0\,.[/mm]
>  
> Somit
>  [mm]\int\limits_{0}^{1}{\frac{\sin^2(x)}{x^2} \ dx} \le \int_0^1 Mdx=M\,,[/mm]
>  
> also existiert das erste Integral in [mm]\blue{(\*)}[/mm]
> rechterhand.
>  
> Wegen [mm]\int\limits_{1}^{\infty}{\frac{\sin^2(x)}{x^2} \ dx} \le \int_1^\infty \frac{1}{x^2}dx=1[/mm]
> (s.o.) folgt auch die Existenz von
> [mm]\int\limits_{1}^{\infty}{\frac{\sin^2(x)}{x^2} \ dx}\,,[/mm]
>  
> d.h. das zweite Integral rechterhand aus [mm]\blue{(\*)}[/mm]
> existiert, und damit existiert das Integral linkerhand von
> [mm]\blue{(\*)}\,.[/mm]
>  
> Zudem können wir mit den obigen Überlegungen
> abschätzen:
>  [mm]\int\limits_{0}^{\infty}{\frac{\sin^2(x)}{x^2} \ dx} \le M+1\,,[/mm]
>  
> was natürlich auch nochmal insbesondere die Existenz
> unseres Integrals zeigt.
>  
> Beste Grüße,
>  Marcel


Bezug
                                                                        
Bezug
uneigentl. integral: Mitteilung
Status: (Mitteilung) Reaktion unnötig Status 
Datum: 14:11 Mo 19.07.2010
Autor: Marcel

Hallo Fred,

> Hallo Marcel,
>  
> damit f stetig ist, sollt man setzen:  [mm]f(0):=1\,.[/mm]

Danke für den Hinweis, es war aber nur ein Verschreiber meinerseits (und das sieht man ja auch an meinen vorhergehenden Antworten; es ist also keine Ausrede meinerseits ;-)). Ich habe es korrigiert.

Auch Danke nochmal für das stets aufmerksame Mitlesen und Mitdenken und für die Korrekturhinweise :-)

Beste Grüße,
Marcel

Bezug
                                                                                
Bezug
uneigentl. integral: Mitteilung
Status: (Mitteilung) Reaktion unnötig Status 
Datum: 14:17 Mo 19.07.2010
Autor: fred97


> Hallo Fred,
>  
> > Hallo Marcel,
>  >  
> > damit f stetig ist, sollt man setzen:  [mm]f(0):=1\,.[/mm]
>
> Danke für den Hinweis, es war aber nur ein Verschreiber
> meinerseits

Das hab ich mir gedacht.


>  (und das sieht man ja auch an meinen
> vorhergehenden Antworten; es ist also keine Ausrede
> meinerseits ;-)).

Das weiß ich. Ich lese Deine Beiträge immer mit Begeisterung und sehe die Kompetenz, die dahinter steht.


>  Ich habe es korrigiert.
>  
> Auch Danke nochmal für das stets aufmerksame Mitlesen und
> Mitdenken und für die Korrekturhinweise :-)


Eigentlich muß man nicht korrigieren. Man könnte auch setzen: f(0):= -4711. Dann wäre f im Intervall [0,1] nur in einem Punkt unstetig, und somit Riemann- integrierbar

Gruß FRED

>  
> Beste Grüße,
>  Marcel


Bezug
Ansicht: [ geschachtelt ] | ^ Forum "Integration"  | ^^ Alle Foren  | ^ Forenbaum  | Materialien


^ Seitenanfang ^
www.matheraum.de
[ Startseite | Forum | Wissen | Kurse | Mitglieder | Team | Impressum ]